Você está na página 1de 65

PROPERTY OUTLINE

I. PERSONAL PROPERTY Introduction: o What is property? In personam v. in rem: In personam: against person In rem: against thing

Bundle of rights (TUPE) T => transfer (or alienate) U => use (the right to use property) P => possession (synonymous to occupancy) E => right to exclude

Establishing property rights: Johnson v. MIntosh: Johnson bought land from Indians; MIntosh bought same land from Congress. Court decided the land belonged to MIntosh because his claim was superior. The Indians could only sell land to government, not individuals, so Johnson bought title from someone who didnt hold it. Discovery: the first party to discover un-owned property gains legal claim to it Conquest: if a party takes over the land and wins it becomes their property

Limitations on property rights: WJF Realty Corp. v. State: Statute: Long Island Pine Barrens Protection Act => you can take but you must compensate Blackstone: property is a bulwark against despotism

Theories of Property: Wild Animals: o o o o Real property rights (rights in land) => real comes from regal Personal property rights (rights in chattel) => movable property Real actions => successful plaintiff recovers actual object of lawsuit Actiones in personam => successful plaintiff recovers damages for wrongs to their property o 2 categories of personal property: Tangible (i.e. animals, furniture, jewelry) Intangible (i.e. bank accounts, stock, debt)

Pierson v. Post: Application of Pierson for wild animals Possession requires Actual control Intent to control

If an animal gets away, its lost Exception for animum rrivertendi animals that return to the same place over and over again Exception for elephant in Texas if there is notice Exception if you have clams on your property that stay in the same place, you may own them whereas you do not have a claim over the deer

Found Property: o When you find something it could have been: Abandoned: Owner no longer wants the property, shown by proof that the owner intends to abandon the property and has voluntarily relinquished all rights to the title and interest Lost: Owner unintentionally and involuntarily parts with property and doesnt know where it is Stolen property found by someone not involved in the theft is lost property Mislaid: Property voluntarily put in a certain place by the owner who then overlooks or forgets where the property is. Differs from lost property because the owner intentionally put it in the place Finder has no rights to the property, rights belong to the owner of the possession on which the property is found against all except the original owner Treasure trove: The finder is the owner only if statutory procedures are followed Belongs to the finder

Coins or money concealed by the owner, includes an element of antiquity, must have been hidden for such a length of time that the owner is probably dead or undiscoverable

Finder has rights to the property against all but the original owner

Benjamin v. Lindner Aviation: Court determines that the money in airplane is mislaid because it was so carefully wrapped and hidden, and because no one loses track of 18K on purpose.

Favorite v. Miller: It was the intention of the loyalists to abandon the property (they wanted to keep the rebels from getting the lead for bullets), in which case Miller, (the finder) would win. But Miller doesnt win, because he was significantly trespassing on the property. As between landowner and the finder, it depends on whether the land was public or private and the length of the duration. (Finders Keepers issue) The more private, and the shorter the duration, the more likely the landowner wins. The more public or the longer the duration, the more likely that the finder beats the landowner. Trivial or insignificant trespass

Bailments: o Bailment: the rightful possession of goods by one who is not the owner; a delivery of property pursuant to a contract o Bailments are at the heart of many commercial agreements o Example: you are a bailee when you rent a car Gratuitous bailment: when you borrow something from a friend

What are the duties of the bailee? Traditional approach: sliding scale of duties that arise based on who benefits from the bailment Sole benefit of bailor: slight care, gross negligence Mutual benefit: ordinary care, ordinary negligence Sole benefit of bailee: extraordinary care, slight negligence

Modern: some modern courts have replaced all this with one standard of ordinary care Consistent to the approach that applies to commercial bailments

Burden shifting: Burden of production: Bailor has burden of production regarding existence of bailment Bailee has burden of production as to the exercise of care

Burden of persuasion: Remains with bailor for the whole time

Allen v. Hyatt Regency Nashville Hotel (Bailment Creation): A bailment for hire was created in this case and when there was nondelivery the Appellee was entitled to the presumption of negligence provided by statute.

Buena Vista Loan & Savings Bank v. Bickerstaff: Liability for negligence in cases of a bailor bailee relationship requires a lack of care commensurate to the hazard. Care less than an ordinarily prudent person would exercise in a similar circumstance results in the bailees liability for negligence. The bailee has the burden of proof to show the goods were handled with due care. Negligence is presumed solely when the goods are not returned or returned with damage.

Shamrock Hilton Hotel v. Caranas: Bailments are of mutual benefit in cases where the bailor is a paying customer and the bailee is in business catering to the general public. Bailees are liable for lost property they have knowledge of and property they could reasonably expect to be contained within the bailed property.

Gifts: o Gift: voluntary transfer of property to another without consideration Property may be transferred gratuitously at death by will or by intestate succession Choses in possession = tangible property Choses in action = debts/intangible property (shares of stock, bank accounts) o By devise = by will By descent = by death without will

Inter vivos: Require a showing of donative intent, delivery, and acceptance Takes effect presently and irrevocably

Causa mortis: gift made in apprehension of death from existing peril/illness

Require a showing of donative intent, delivery, and acceptance Takes effect presently Revocable under certain circumstances (express revocation, recovery from peril/illness, or outliving donor)

Must have contemplation of death Intent must be to make a present gift, not a gift in the future Anticipation of death induces someone to give gift Delivery sometimes has a higher requirement: Courts sometimes say that if the good was already in possession of the donee, there wasnt sufficient delivery

Aunt IDA: Intent: intent to donate by the donor Delivery: can be substituted with a deed or the equivalent of delivery Modern methods of delivery Actual physical possession Constructive delivery of the means of acquiring possession o Actual delivery has to be impractical or impossible (small jewelry box v. chest of drawers. Intent of giving someone merely the key may not be enough to signify intent) Symbolic Instead of giving the actual good, you give a symbol of the good o Actual delivery must be difficult, but need not be impossible 3
rd

party asking a 3rd party to pass it on to another party o Has to be given to a party of the other side

Acceptance: by the donee, very important

Irvin v. Jones: Woman bought CDs for self and nephews (JT with right of survivorship), she cashed them before time, they sued because they thought they were theirs Delivery is indispensable element, and here no delivery because intent was to hang onto them in case need them- testamentary intention There must be intent to give irrevocably and immediately coupled with actual delivery, also acceptance

Delivery is evidence of intent, also could be delivery to third party to hold for intended donee, third party holds the gifts as trustee.

Gruen v. Gruen: A valid intervivos gift was made as the donor intended to make a gift to his son, only constructive delivery was needed as actual delivery of the painting to the Plaintiff would have defeated the donors intent to retain a life estate in the painting and acceptance is deemed presumed as it is a benefit to the donee.

In re Estate of Gladowski: Joint bank account generally presumed as a gift, but can be overturnedhere not gift -meant to go to all children, her name as liaison- and these were the only assets -rebuttable presumption that depositor of funds intended a gift to survivor -inter vivos not contemplating death

Braun v. Brown: Gave means to access the box, delivery is manifestation of intent -had witnesses, etc.- all indicates the intent (question of fact)- just need words and circumstances, gave her key, witness, lots of other gifts

Heiman v. Parrish: Engagement ring purchased by man for woman, man ended relationship, no fault stipulated Conditional gift? Most say yes, but also fault/no fault approach -no fault approach- donor wins every time -fault- person who wasnt at fault for ending relationship gets the ring, if mutual then donor -inter vivos giftwoman -K theory- to see who breached the K and that determines the outcome- dont favor breachers (detrimental reliance damages etc) -NO FAULT- fault too hard- modern trend

II. LANDLORD-TENANT Introduction: Leases Creation and Classification: o Landlord-tenant: Traditional landlord-tenant legal principles were developed on the basis of a conveyance theory that gave lessees a limited possessory interest in the land, and consequently were considered part of property law Modern landlord-tenant law includes many contractual principles => often collide with property law

Statutory modification of common law uniform residential landlord and tenant act

Leases creation and classification: Is there a contract, is there a term on point, is it enforceable? Creation statute of frauds: English statue of frauds => required written evidence for certain lease agreements, contracts, etc.; oral leases over 3 years had force and effect of leases at will only Most states have adopted this, modern statute of frauds limit duration to 1 year Gee v. Nieberg: General Rule: Where the original k is required by the statute of frauds to be in writing, it may not be modified or varied by a subsequent oral agreement, but oral rescission of an executory contract is valid even if the contract was required to be in writing under the statute of frauds.

Classification by duration: Tenancy for a fixed term (estate for years): No common law limitation on the duration of a tenancy for a fixed term => some state statutes set a maximum Terminates automatically at the end of the term, landlord need not give advance notice Periodic tenancy: Lease giving lessee possession of property for successive identical periods of time => month-to-month is most common May be created by express agreement or imposed by operation of law Of potentially indefinite duration because it is automatically renewed for successive periods unless one party gives notice to terminate Common law rules on notice to terminate: o o 6 months notice for year-to-year tenancy Notice equal to the length of the period where the period is less than one year o Date for termination must be at end of period

Tenancy at will: May be terminated at election of either party Common law: no advance notice required => many state statutes provide for notice to terminate May be created by express agreement, or be created because agreement is neither fixed term nor periodic

Tenancy at sufferance (holdover tenant): One who remains on leased property after termination of lease Absent lease provision to the contrary, landlord has option to treat a tenant at sufferance as either a trespasser or new tenant

Tenants Right to Possession and Quiet Enjoyment o Possession at the commencement of the lease: American rule (minority): the lessee has a right to possession, but absent an explicit covenant, the lessor has no duty to deliver possession English rule (majority): landlord is required to deliver actual possession of the premises to the tenant when the lease term begins, in addition to the legal right to possession o Quiet enjoyment during the lease term: Implied in leases in virtually all states, and is expressly included in many written leases Breached when tenant is evicted by the landlord, by someone acting under the landlords authority, or by someone asserting paramount title Actual eviction: Tenant does not have to pay rent if the landlord interferes with the tenants possession (quiet enjoyment) of the premises Camatron Sewing Machine, Inc. v. F.M. Ring Associates, Inc.: Leased part of a building for 10 years had a provision that stated could make renovations etc. without it being eviction, wanted to make changes that took away space from leased property It is partial actual eviction -not authorized to do this under the lease Tenant has sole and exclusive right to undisturbed possession during the term of the lease and the landlord

has no right to take possession of a part of the demised premises to the exclusion of the tenant -zealous about tenants right to possess Constructive eviction: Landlord must breach some duty to the tenant; that duty must be found outside the covenant of quiet enjoyment which imposes no specific obligation on the landlord, requiring only that he not interfere with tenants possession Automobile Supply Co., v. Scene-In-Action Corp.: Long-term commercial arrangement, heat had been out over several occasions and temperature really low Must be considered grave and permanent breach of duty, clearly indicating the intention of the landlord to deprive the tenant of the longer beneficial enjoyment of the premises in accordance with the terms of the lease Constructive eviction is a defense not to pay rent

Net Realty Holding Trust v. Nelson: Loitering case Generally, its not the landlords responsibility/duty if the nuisance is a third party

Condition and Maintenance of Leased Premises o Traditional approach: Landlords duty: Traditional view => caveat emptor => tenant takes premises as is and landlord has no duty to repair => tenant could inspect and make any necessary repairs Even traditional common law recognizes exceptions for (1) short term leases of furnished dwellings, (2) leases of buildings under construction, and (3) cases with fraudulent misrepresentation Tenants duty: Tenant has duty not to commit waste Waste = material change to the property, even if it enhances the value of the property Failure to act = permissive waste Act or commission = voluntary waste

Temporary changes are probably not material (no magic formula) o o o o Did not affect the structure Long-term lease Cost of changes a percentage of total value Indications of the intent of the parties in the agreement o Least cost avoider principle

Suydam v. Jackson: No provision that the landlord has to keep the roof repaired. Should L or T win? At common law, it depends on whether the building is destroyed or injured. Court held that this applied only to catastrophes in a statute. Tenant is responsible for maintenance

Sparkman v. Hardy: Tenant lease for 10 year, with option to stay for 15 more years, 7 years in, landlord refused to accept rent because tenant makes changes to premises, put up some partitions, changed front entrance Any material change in the nature or character of the buildings made by the tenant is waste even if value of enhanced (affect walls, foundation, roof The longer the lease, the more change is allowed before it is considered waste

Implied warranty of habitability: Virtually all states have rejected caveat emptor for leases of residential property Warranty of habitability is implied => bare living requirements Wade v. Jobe: Failure to supply water or heat, or violations involving safety and health will often sustain a tenants claim for breach of warranty of habitability. If a claimed defect has an impact on the health and safety of the tenant a code violation is not necessary to establish a breach. Davidow v. Inwood North Professional Group-Phase 1:

An implied warranty of suitability means that at the inception of the lease there are no latent (possible/idle) defects in the facilities that are vital to the use of the premises for their intended commercial purpose AND that these essential facilities will remain suitable. Businesses are usually treated differently than residential properties, but court likened doctors office to residence

Express covenant to repair: Covenant by landlord: Lessor is not required to rebuild on substantial destruction by fire unless he is obligated to do so by the terms of the lease Covenant by tenant: In the commercial area, tenants often take on covenants to repair and may undertake exceedingly broad expectations Amoco Oil Company v. Jones: Fire broke out and destroyed gas station Common law rule: D would be obligated to rebuild. Based on conveyances of interest in land, inappropriate now Modern trend: some courts have held that one cannot repair what does not exist (distinction between repair and restore). Courts look to the intent of the parties at the time of the lease execution and the plain meaning of the language used.

Assignment and Sublease o Objective test: if entire remainder of lease is transferred it is an assignment; if only part of lease is transferred it is a sublease o Privity of contract: Sue over personal covenants. Parties in privity of K can sue for anything one can sue for in a K dispute. Covenants that do not run with the land must be expressly assumed by T2 in order for T2 to be bound. An express covenant to pay rent can follow someone in privity of K. L would sue in K after suing in estate. o Privity of K only passes when another party expressly assumes the terms

Privity of estate: Sue over things that run with the land.

Only 1 party can have privity of estate at one time. When it passes from one to another, it's gone.

Privity of estate does NOT pass when a T subleases, but does pass when a T assigns a lease.

If the landlord consents to the first assignment or sublease, it is assumed that the landlords assent to further assignments or subleases

Limits on Assignment and Sublease L's generally prefer assignment over sublease. And definitely prefer for T2 to expressly assume T1's K obligations. Old CL Default Rule: T freely able to assign or sublease, without silent consent clause. Modern Rule: don't like limits on assignment, but will look to a reasonable interpretation of party's intent. Rationale: Don't like limits on free alienation; prevents full utilization of land. If T1 has some special skill or something unique about their ability, court is more likely to find an implied limit on assignability.

Rowe v. Great Atlantic & Pacific Tea Co., Inc.: Restrictive covenants will be enforced if the landlord relied on a special skill of the lessee; must be more than just relying on the month to month rent In this case, percentage clause is not enough, did not generate much, if any, revenue. Threshold amount was lower than regular sales. Also, new lease was improvement on old lease.

Julian v. Christopher: Consent cannot be withheld unreasonably, unless a freely negotiated provision in the lease gives the landlord an absolute right to withhold consent Traditional property rules favor the free and unrestricted right to alienate interests in property

First American National Bank of Nashville v. Chicken System of America, Inc.: Anyone who gets an assignment has to pay rent by privity of estate, can get rid of it by dumping, but if the assignee were contractually obligated to pay rent, then they are obligated.

Termination of the Landlord Tenant Relationship o Methods of terminating leases:

Surrender: T surrenders and L accepts Statute of frauds usually applies => if lease was in writing than surrender must be in writing too

Abandonment: Occurs when T: Vacates without justification Lacks present intent to return Defaults in the payment of rent

Ls traditional options: Leave premises vacant and sue T for accrued rent Mitigate damages by re-letting and sue T for difference => most jurisdictions have moved towards requiring this Terminate the lease o Modern laws make it difficult for L to terminate Ts lease o Statutes in most states protect T from retaliatory eviction

Self help eviction: Minority approach => L can retake possession from T if T doesnt leave after being evicted => viewed as a feudal relic in modern times

Summary eviction: State procedures vary; generally have a wrongful detainer statute providing for expedited eviction procedures Assurance of adequate housing and definition of landlord-tenant relationships are legislative, not judicial, functions Some states have separate housing courts to deal with these types of claims

Retaliatory eviction: Landlord can evict for any legal reason, or no reason at all may not evict for retaliatory reason though

Illegal activity by tenant: Traditional proceeding that was available to the landlord was an action called unlawful detainer Preceded eviction

Common law: If tenant uses property for unlawful purposes landlord can sue for damages but couldnt terminate lease

Modern law: If tenant uses property for illegal purposes, landlord can terminate. K provisions (or statutes) that allow termination for illegal activity will be enforced in court.

III. INTERESTS IN REAL PROPERTY Estates in Land o o o Right of present possession => present estate / present possessory estate Right to possession at a future time => future estate / future interest Classifications: Freehold Non-freehold: o Now called leases Historically used primarily for agricultural purposes

Theoretical difference: presumed prestige Practical difference: courts

Four groupings: Fee simple absolute: To A and his heirs Property now belongs entirely to A And his heirs became mandatory, absence meant no fee simple absolute was conveyed Eliminated in modern times A takes estate of indefinite duration => A is the purchaser, to A are words of purchase => and his heirs are words of limitation indicating estate can be inherited Defeasible fees: Fee simple determinable: To A for so long as A uses the property for [agricultural purposes] Created by the transfer of a fee simple to endure until the occurrence of a given event Until, as long, while If the event occurs the estate automatically ends

Common law => an estate in the transferor was created simultaneously with the creation of a fee simple determinable

Fee simple subject to a condition subsequent: To A, but if A fails to use the land for [agricultural purposes], O may enter and recover the land Closely related to fee simply determinable except for several important differences: o Happening of a certain event gives the transferor the right to enter and terminate the estate o Future interest is simultaneously created in the transferor o Becomes possessory only when the condition is met and the owner of the right of entry takes appropriate steps to terminate o Markedly different with respect to termination

Fee tail: To A and the heirs of his body Least important in modern law Origins => when people wanted to pass their land to future generations without end => assure present owner maximum use of property => Statute De Donia Conditionalibus Kept land in family perpetually Disadvantage: Restrictive alienation => land may not be used in most productive way No longer intentionally used anywhere in the U.S.

Life estate: To A for life Not typically an inheritable estate May end before the transferees death => may be defeasible (i.e. as long as youre my widow), or may be merged with larger estate Variation: pour autre vie, estate for the life of another Generally, life tenant has the absolute right to possess the property and enjoy its income

Problems arise if: Life tenant depletes estate (i.e. by cutting down timber) => doctrine of waste was established because of this Question of who is responsible for paying tax, etc. on property may arise

Non-freehold estates (leases): Differs because it is not real property, regarded as chattel real Could not be subject to real actions available for the protection of freehold estates Forms: Tenancy for a fixed term Periodic tenancy Tenancy at will Tenancy at sufferance

Future Interest Classification o o Future interests: estates which become possessory at a future time Five types: Possibility of reverter: Future interest created by a fee simple determinable Becomes possessory only on the termination of the fee simple estate Ex. O conveyed to A for so long as A uses the land for agricultural purposes Freely alienable (you can sell it), devisable (you can transfer it by will), and descendible (you can pass it to heirs without a will) in most jurisdictions Not subject to rule against perpetuity

Right of entry: Interest retained by a transferor who has created a fee simple subject to a condition subsequent Future interest which becomes possessory only on the termination of the preceding fee simple estate Most require right of entry to be explicitly spelled out, court may find it by implication in some circumstances though Ex. O conveyed to A in fee simple, but if A ever fails to use the land for agricultural purposes, O may enter and recover the land

Generally devisable and descendible Usually transferable inter vivos only when coupled with reversion

Reversion: Follows life estate, fee tail, or term of years That interest in remaining in one who transfers a lesser estate than the estate that person owns No need for the transferor to spell out the reversion in the instrument creating the present interest Law implies reversion whenever one transfers a lesser estate than that owned Fully alienable, descendible and devisable Not subject to the rule of perpetuities

Remainder: Can only follow fee tail, life estate, or leasehold Must be created at same time as the preceding supporting estate Future interest created in the transferee that can become a present interest upon the expiration of all prior interest simultaneously created and cannot divest any interest except interest left in the transferor Only becomes possessory at the natural expiration of the prior freehold estate Must become possessory immediately upon the natural termination of that estate => can be no gap Can only be created at the same time as the preceding supporting estate Two types: Vested: o Transferred to a born and ascertained person, AND there is no condition precedent to the taking o Forms: totally vested, vested subject to partial divestment, vested subject to complete divestment o o Alienable, descendible and devisable Examples: O conveyed to A for life, remainder to B => B has totally vested remainder

O conveyed to A for life, remainder to the children of B (a living person) => at time of conveyance B has one child, C-1 => C-1 has vested remainder subject to partial divestment in favor of other children who may be born before the remainder becomes possessory on As death

O conveyed to A for life, then to B, but if B fails to marry before A dies, then to C => remainder to B is vested subject to complete divestment

Contingent: o Freely alienable, descendible, and devisable in most jurisdictions o Either: (1) subject to a condition precedent, or (2) created in an unborn or ascertained person o **Condition subsequent => no remainder

Examples: O conveyed to A for life, remainder to the heirs of A => As heirs cannot be ascertained until As death => heirs remainder is contingent O conveyed to A for life, remainder to the first child of A => A has no children => contingent O conveyed to A for life, then if B marries before A dies, to B => contingent

Executory interest: A future interest capable of creation in the grantee that cuts short the preceding estate. May trigger the Rule Against Perpetuities. Shifting executory interest: cuts short the interest of another grantee Ex: Olivia conveys Orangeacre To Arnold and his heirs, but if Claire gets married, to Claire and her heirs. Arnold has a fee simple subject to Claires shifting executory

interest. Claires shifting executory interest will become possessory if she gets married. Springing executory interest: cuts short the interest of the grantor Example: Olivia conveys Yellowacre To Arnold if he becomes a lawyer. Olivia has a fee simple subject to Arnolds springing executory interest. Arnolds interest will cut short Olivias interest and become possessory only if he becomes a lawyer. Common Law Rules Limiting Future Interests o Rule in Shelleys case: At common law, if the same instrument created a life estate in A and gave the remainder only to As heirs, the remainder was not recognized, and A took the life estate and the remainder. Most jurisdictions have abolished this rule. Effectively abolished Fee Tails: all in A, and land now alienable.

How to beat: You beat the rule by saying O conveys to A for life, one day thereafter to As heirs. A has a life estate. A dies, and the heirs have a springing executory interest, wresting the interest out of the transferor

Does the rule apply in the following cases? O conveys to A for life, then to B for life, then to Bs heirs. B has a vested remainder. But the rule merges Bs and Bs heirs interests to form a Fee Simple Absolute. O conveys to A for life, then to Bs heirs. Bs heirs have a contingent remainder. Because of difference between A and B, the rule does not apply.

Doctrine of worthier title: A remainder in the grantors heirs is invalid and becomes a reversion in the grantor. E.g., if A grants Blackacre to B for life, then to the heirs of A, B has a life estate, and A has a reversion, not As heirs. This doctrine is generally treated as a rule of construction (i.e., it does not apply if intent to create remainder in heirs has been clearly manifested). It applies only to inter vivos transfers (not wills), and only if the word heirs is used. It is not recognized in many jurisdictions. O to A and then to Os heirs

Before Rule: A has life estate. Os heirs have a contingent remainder and executory shifting interest.

After rule: CR and ESI eliminated. O gets a reversion. Therefore, O can sell the reversion, i.e., alienate that interest, leaving Os heirs with nothing.

Again, O gets the whole package.

Destructibility of contingent remainders: At common law, a contingent remainder was destroyed if it failed to vest before or upon the termination of the preceding freehold estate. E.g., A conveys to B for life, then to C if she reaches age 21. If B dies before C reaches 21, Cs remainder (which is contingent on her reaching 21) is destroyed. A, or his successors, then has a reversion interest. Under the doctrine of merger, A can merge his present life estate with his newly acquired reversion to create a present possessory fee simple absolute.

Rule against perpetuities: No interest in property is valid unless it must vest, if at all, not later than 21 years after some life in being (measuring life) at the creation of the interest. If there is any possibility the interest might vest more than 21 years after a life in being, the interest is void. In other words, An interest is void if there is any possibility, however remote, that the interest may vest more than 21 years after some life in being at the creation of the interest. If a situation (even highly unlikely or unrealistic) can be imagined in which the interest might not vest within the perpetuities period, the interest is void. Principal Target: vesting of contingent remainders But applies to both contingent remainders and executory interests (i.e., interests of transferee --- remainder and executory interests.) NOT applicable to Rights of Entry and Possibilities of Reverter.

Does the rule apply in the following cases? O devises to A for life, then to B if B survives A. Rule does not apply; A has a life interest and B has a contingent remainder in fee simple absolute. Os heirs have a reversion interest (if B does not survive A).

O conveys to A so long as the property is used for commercial purposes, then to B. Rule against Perpetuities renders Bs contingent interest void because it is unknown how long the property will be used for commercial purposes.

O conveys to A for life, then to B. O is 93 and in poor health, A is 10 years old. Rule does not apply and ages are not important; A has a life estate and B has a vested remainder in fee simple absolute.

O conveys to A for life, then to As first child to reach 25. At time of conveyance, A has one child, age 24. Rule applies and renders the interest in As children void because As first child could die and A could die, making it conceptually possible that the interest might not vest within As life plus 21 years. A has a life estate, and As first child has a contingent remainder.

O conveys to A upon As marriage to B. Rule does not apply because the uncertainty will be resolved within As life plus 21 years. O has a fee simple subject to As springing executory interest; A has a springing executory interest in fee simple absolute.

Designed to prevent undue accumulations of land and wealth in the hands of a few

Adoption of rule caused by donative transactions (Ex. Wills, deeds, etc. in which owner had large amounts of capital to pass to his heirs)

Doesnt further free alienation to the extent it once did Several developments have undercut the Rules effectiveness: Usually geared to the duration of lives in being at the creation of the interest => average life spans have increased Society which the rule was developed to serve no longer exists => pace of change All reforms of the Rule undertaken recently tend to validate longlasting contingencies

One common modification => wait and see period => RaP is determined to have been violated or not by taking into consideration events which occur after the period fixed by the rule has commenced => saves arrangements that would otherwise by invalid under the common law Rule

Some states have completely abolished the Rule

Limited to future interests (a) in third parties, where (b) the third party is unascertained or there is a condition precedent to the interest being vested Contingent remainder Executory interests Vested remainders subject to open

Concurrent Interests o Types: Tenancy in common: Tenants concurrently own the same or different percentage interests in the property, but each is entitled to possession of the whole. Interests are alienable, devisable, and inheritable. When a tenant dies, his interest passes to his heirs. Today, multiple grantees are presumed to take as tenants in common, not as joint tenants. Joint tenancy: A joint tenancys distinguishing feature is the right of survivorship. When a joint tenant dies, the interests of the remaining joint tenants are enlarged proportionately. It requires unities of time, title, interest, and possession. Joint tenancy can exist only where cotenants acquire their interests at the same time from the same instrument, and the interests are identical and undivided. A joint tenant may freely convey his individual interest and sever the joint tenancy. If this occurs, a tenancy in common results between grantee and remaining tenants. Tenancy by the entirety: A marital estate akin to joint tenancy, limited exclusively to married couples. It includes a right of survivorship and cannot be severed unilaterally. Only death, divorce, mutual agreement, or execution by a joint creditor of both the husband and wife can sever a tenancy by the entirety. An individual spouse cannot convey or encumber tenancy by the entirety property. If A and B are conveyed land as tenants by the entirety but are not married, courts will either interpret the arrangement as tenancy in common or a joint tenancy. o Creation

Presumption of Tenancy in Common unless clear intention to the contrary is shown

In re Estate of Michael => court interpreted a deed that conveyed property to two couples as tenants by the entireties as a tenancy in common between the two sets of married couples because there was no clear language establishing a joint tenancy. Therefore, the court held that when both members of one couple died, they were free to convey their interest in the property to their son by will.

Rights of Creditors: With a tenancy by the entirety, there are three approaches to the rights of creditors to one partys interest: Interest of debtor spouse in the estate may be sold or levied upon for his or her separate debts, subject to the other spouses contingent right of survivorship. Attempted conveyance by either spouse is void, and estate may not be subjected to the separate debts of one spouse only during marriage. In Sawada v. Endo, the court followed this approach and held that a married couples conveyance of their property to their sons ostensibly to avoid husbands judgment creditors was not done in fraud. Contingent right of survivorship of either spouse is attachable by creditors during the marriage.

Rights and Obligations among cotenants Rent Liability among tenants in common: If a co-tenant is excluded from use of the property by the other tenants, the excluded tenant has the right to collect rent for her portion of the property from the exclusive tenants. If a co-tenant rents out his portion of the estate, he must distribute rent proportionately to all other tenants. In Graham v. Inlow, the court found that a stepmother did not exclude her stepdaughter from possession of the property and therefore was not liable for rent payments to the stepdaughter. Improvements to property:

A co-tenant can receive enhancement value of the improvement to the property; proper measurement is the difference in value between the value of the land without improvements and the value of the land with the improvements in their then condition. (Graham v. Inlow).

Ouster: If a tenant holds property as a result of ouster of the other tenant, he is liable for rent to the other tenant. Ouster must be manifested or communicated to the other tenant. In Coggan v. Coggan, an ex-wife was not granted of the rental value of a building which she and her ex-husband held together as cotenants because the husband used the building for his own benefit, did not receive profits therefrom, and most importantly, he did not oust the wife from possession through his manifestations or communications.

Termination: Effect of divorce upon tenancy by the entirety: Majority view: Divorce nullifies joint tenancy and arrangement becomes a tenancy in common because intent of parties is most likely NOT to hold the property together. Also because tenancy in common is presumed absent a clear expression to the contrary. Minority view: Divorce does not affect title of parties to the property, and arrangement becomes a joint tenancy. This was the view adopted in Shepherd v. Shepherd, which found that the ex-husband was entitled to the house upon the death of his ex-wife by right of survivorship. The wifes estate argued that the arrangement became a tenancy in common upon divorce, and therefore her heirs were to receive of the estate. Effect of divorce upon joint tenancy: A divorce decree that is silent with respect to property held jointly with right of survivorship does not destroy survivorship provisions; the parties must have a clear INTENT to partition the tenancy. In Porter v. Porter where a second wife sued for of the interest in a home held by her husbands 1st wife, the court

held that the divorce of the 1st marriage did not destroy the joint tenancy arrangement. When the Mr. Porter died, his first wife became the sole tenant by right of survivorship. Joint Tenancy can be severed by executing and recording a unilateral selfconveyance: Most jurisdictions allow a joint tenant to sever a joint tenancy by either conveying ones interest in an estate to oneself, or by using a strawman. Courts hold that it is important to show intent to sever, and that severance must be recorded to demonstrate intent and give notice to the other party. In In re Estate of Knickerbocker, the court upheld a wifes unilateral self-conveyance on the grounds that using a strawman went against public policy (it is wrong to require that a party do indirectly what she can do directly). Effect of Joint Tenants conveyance of lease: When a joint tenant leases his interest in property to a third party, courts do not have a uniform approach to deciding what effect the lease has on the joint tenancy. Some courts hold that a lease does not amount to a severance of the joint tenancy. Under this view, if the joint tenant dies, the lease becomes null and the tenants interest goes to the other joint tenant by right of survivorship. This was the view adopted by the court in Tenhet v. Boswell. Some courts hold that a conveyance by lease amounts to a severance. Under this view, if a joint tenant dies, his interest in the estate goes to his heirs upon his death. Some courts take another stance and hold that a conveyance amounts to a temporary severance. Co-owner has the right to partition land: In Coolidge v. Coolidge, the court held that mere contemplation of partition is not sufficient to show that there was an implied agreement not to partition land. Even if there is an express agreement not to partition land, courts will not uphold the agreement if it is unreasonable. Prohibiting partition runs counter

to public policy because it imposes unreasonable restraints on alienation. Property may be partitioned according to acreage or monetary value, but physical partition is preferred unless it would result in prejudice to the owners: Whether partition would result in prejudice is determined by whether after partition, the value of the share of each portion of land together is less than the value of the whole. A court can only set aside the commissioners ruling on partition if the division is disproportionately unequal to the % of land each party owned. In Von Behren v. Oberg, the court held that partition was not disproportionately unequal when the party owning a interest in land was given 120 acres, and the party owning interest was given 50 acres in partition. Effect of a joint tenancy or tenancy in common on partition: In a tenancy in common arrangement, when land is partitioned and one tenant has contributed more to the improvement of the land or more to the purchase price, that tenant will get more in partition. In a joint tenancy, the court presumes equal contribution, but this presumption can be rebutted. o Condominiums: Arrangement: A condo arrangement involves both a fee simple absolute and a tenancy in common. A condo tenant owns his individual unit but is a tenant in common with respect to the actual building and the grounds of the condo. A condo owners association insures that common elements are maintained. Some courts have treated the relationship between the individual and the owners association as a landlord-tenant arrangement. Conversion: Many jurisdictions regulate conversion through legislation. Regulatory measures include conditioning conversion upon approval by a % of the tenants, giving tenants advance notice and right of first refusal, and moratoriums on conversion. Time-sharing:

Arrangement is either as a tenancy in common, whereby individuals get an undivided fee simple interest in a unit and make separate agreements for the right to use a set time period each year, or as an interval ownership. In interval ownership, each owner has a tenancy for a fixed term, and an undivided interest in a vested remainder, which becomes possessory at the end of the specified number of years.

Cooperatives: In an apartment building, members purchase the shares of stock assigned to the apartment to be occupied. When stock is purchased, the purchaser receives a long-term proprietary lease. Purchaser has an ownership interest in the entire project and the right to occupy a particular apartment as long as the stock is retained. Members are financially interdependent, so if several members fail to pay rent, the blanket mortgage cannot be covered. In some jurisdictions, summary eviction can be used for members who do not pay rent, but other jurisdictions reject this approach because there is no clear line between ownership and leasehold.

Marital Interests o o A married person may have rights in the property owned by his/her spouse At common law (jure uxoris, curtesy, and dower) By marriage a husband and wife are one person in law The husbands bundle of rights in his wifes property was termed an interest jure uxoris, whereas the wife was said to be under the disability of coverture Courts of equity reduced rigidity => recognized wifes separate estate in her property 19th century => many states started enacting Married Womens Property Acts => provided real and personal property of a wife should continue as her separate estate Husbands interest jure oxeris lasted only for their joint lives => husbands right of curtesy vested on the live birth of a child Common law gave wife dower interest in the land of her deceased husband => arose at time of marriage and remained until husbands death => did not require the child of the marriage be born alive => dower rights remained attached to husbands land even after he conveyed it away =>

did not attach to equitable interests in land => creditors of deceased husband could not reach wifes dower interest Common law interests of curtesy and dower have not survived intact Many states have opted for a statutory elective share for the surviving spouse => measures designed to prevent a decedent from suing a will to exclude a surviving spouse => surviving spouse usually gets 1/3 to 1/2 of estate, fee simple o Community property: 9 states reject the common law approach to marital property in favor of the civil approach => marriage is treated as an equal partnership Property acquired during the marriage is community property, property brought into the marriage and property received by one spouse by gift/descent/devise is separate property Difficult to distinguish between community and separate property Community property is a statutory creation Generalizations of community property: Conveyance: both spouses must join in conveyance of community property, but each spouse can convey separate property Management: community property is managed equally, separate property is managed individually Claims of creditors: creditors of either spouse can reach community property Divorce: absent a property settlement, community property is equally divided Death: at the death of one spouse, community property remains with surviving spouse, other passes by will or intestate along with decedents separate property Waiver: may waive community property rights by either antenuptial or prenuptial agreements o Types of community property Homestead Applies to property which family uses as a residence. Exempts that property from the creditors of the owner. Because of concerns about bankruptcy, Congress passed legislation which imposed new restrictions on the size of a homestead exemption you can claim

Non-Possessory Rights and Interests: o Easements Right to use or limit the use of another persons land for a special purpose (e.g., to lay pipe, to access a road or lake), but not the right to possess that land. Can be affirmative => the holder is entitled to make affirmative use of the servient tenement, or negative => entitles the holder to compel the possessor of the servient tenement to refrain from engaging in an activity on the servient estate. An easement appurtenant benefits land owned by the easement holder and is viewed as attached to that land: For an easement to be appurtenant, there muse be two tractsthe dominant tenement (the estate benefited by the easement), and the servient tenement (the estate subject to the easement right). An easement in gross gives the holder a right to use the servient tenement independent of his possession of another tract of land; i.e., the easement benefits the holder personally rather than another parcel of land. An easement in gross for the holders personal pleasure (e.g., right to swim in the pond on Blackacre) is not transferable, but one that serves an economic or commercial interest (e.g., right to erect billboards on Blackacre) is transferable. Creation of Easements: Express Provision: Any easement for more than one year must be in writing and signed by the holder of the servient tenement. In Corbett v. Ruben, there were two parcels of land owned by one person until 1964. In 1962, the owner created an easement for parking upon parcel 1 as an appurtenance to parcel 2. This easement was void because an owner cannot create an easement for his own benefit. In 1964, the 2 parcels were conveyed to 2 different parties, and the seller expressly created the same easement that he tried to create in 1962. Plaintiff was successor to parcel 1 and argued that the easement was void. The court decided that the defendant was entitled to the easement. The following rules were established:

Easement can be granted expressly as long as the intention to grant is manifest on the face of the agreement.

Easement is presumed to be easement appurtenant. This furthers the public policy of using land efficiently and prevents parties from being unnecessarily deprived of a benefit. The test of appurtenancy is whether the easement is a useful adjunct to the property.

Courts may presume an appurtenant easement was intended to terminate when the purpose it was created for can no longer be served.

An easement appurtenant passes with the dominant estate to the grantee even though it is not specifically mentioned.

Express Reservation: Arises when a grantor conveys title to land, but reserves the right to continue to use the tract for a special purpose. Many courts hold that a grantor cannot reserve an easement to benefit a stranger (stranger to deed rule), but a number of courts have rejected this rule in favor of effectuating the intention of the transferor.

Implication: Created by operation of law rather than by a written instrument. Four elements must be shown to establish an easement by implied reservation: o o Common ownership Permanent and obvious servitude was imposed on one part of an estate in favor of another during the unity of title o o Apparent and continuous use, i.e., continuity Easement is reasonably necessary for the enjoyment of the dominant part Easement Implied by necessity: An easement by necessity arises when an estate is severed, leaving the dominant estate without a means of access. The owner of

the servient parcel has the right to reasonably locate the easement. The party asserting the easement must prove that it is strictly necessary for the enjoyment of the property, and it is not dependent upon the existence of any established route or quasi-easement prior to the severance of the estate. Terminates when necessity ceases. o Agreements whereby access to land is forfeited will be carefully scrutinized by courts in view of public policy of promoting the efficient use of land. o In Whitt v. Ferris, the court held that where language in a deed is ambiguous, intent of the parties should be looked at. Courts will allow parties to create deeds that expressly exclude easements. However, even if the language looks like an easement is not included, a court can still find a way of necessity. In this case, one of the defendants had an implied easement by way of necessity because he was conveyed a parcel of land which had no outlet to a public road except over the disputed land. The court held that his easement was limited to the purpose for which it was created, and therefore only the graded roadway could be included in the easement, and not the extra 40-feet of grassy area on the side of the roadway. Easement implied from a quasi-easement: An easement may be implied if prior to the time the tract is divided, a use exists on the servient part that is reasonably necessary for the enjoyment of the dominant part and a court determines that the parties intended the use to continue after division of the property. It is called a quasi-easement because an owner cannot hold an easement on his own land. o Elements:

Common ownership at time when estate was severed

Common owner's use of part of the land to benefit another part (existing at time tract was divided) was apparent and continuous at the time the tract is divided. Apparent means that a grantee could discover the existence of the use upon reasonable inspection. A nonvisible use may still be apparent if surface conditions or the like would put a reasonable person on notice of its existence.

Land was actually transferred At severance, it was reasonably necessary to continue the preexisting use for the benefit of the dominant estate. Some courts require that strict necessity be shown, but most courts require only that reasonable necessity be shown.

Reasonable Necessity exists where great effort and expense would be necessary if easement did not exist. In Schmidt v. Eger, P leased property to D and during the course of the lease, plaintiff placed a ditch on the leasehold. D later exercised his option to buy the land and told P that he was going to level the property that contained the ditch, and P sought an injunction to prohibit D from interfering with the drainage ditch, claiming that the ditch represented an easement by implied reservation. The court held that the continuity element was satisfied, as well as the test for reasonable necessity (it would cost far too much to create a new drainage

system); however, P failed to satisfy the element of apparent and prior use because the drain was not created, and thus not apparent, until after the date of severance (which was the day when the leasehold was created). Therefore, there was no implied easement. o Reasonable necessity must be more than convenient and beneficial. In Whitt v. Ferris, the court held that if an owner of land can use another way, he cannot claim by implication the right to pass over that of another to get to his own. In this case, 2 of the Ds had access to a main road without having to use the disputed parcel; one defendant only needed to create a road between his front and back lots, and the other defendant only needed to cut down a few trees in order to have an outlet to the main road. Therefore, the court found that it was not reasonably necessary to burden plaintiffs lot when use of the disputed parcel was merely convenient for the defendants. Implied from Plat Map: Where a conveyance of land describes the property by reference to a plat upon which a street is depicted, an easement is implied over the entire street for the benefit of the lots represented in the plat. In Whitt v. Ferris, the conveyances were made after the plat was vacated, and therefore no private easements existed when the lots were sold. Subdivisions: There are two viewpoints adopted by different jurisdictions on the extension of easements in subdivision plat: o Easement extends to all streets delineated on subdivision plat.

Easement extends only over the street on which an owners property abuts and to the next intersecting street that permits access to a public road.

In De Ruscio v. Jackson, the court adopted this approach and held that plaintiff was entitled to an easement over two streets.

Prescriptive: Scope of it is parallel to the use that established the easement in the first place. Requirements for easement by prescription: o o Continuous actual use Open and notorious - known or knowable to party against whom the claim would exist o Must be adverse

Scope of easements: Easements created by general grant or reservation are not affected by a reasonable change in the use of the dominant estate where easement is not limited. In Hayes v. Aquia Marina, the court held that because there was no limit on the dominant estates easement to use a road on plaintiffs property to gain access to the marina, the Marina was allowed to expand in size. The court found that no additional burden would be placed on the servient estate because traffic would not increase significantly when the marina increased in size. The court also found that paving the road was a reasonable change to the easement. Different use of easement not allowed: No use may be made of an easement which is different from use established when the easement was created. However, absent specific limitations in the grant, courts assume the easement was intended to meet both present and future needs of the dominant tenement (e.g., easement may widen to accommodate new, wider cars). Use that imposes an additional burden on servient estate not allowed. E.g., if the dominant parcel is subdivided, the lot

owners will not succeed to the easement if to do so would unreasonably overburden the servient estate. Dominant estate holder has right to improve an easement so long as the improvement does not unreasonably increase burden on the servient estate. In Hayes v. Aquia Marina, paving the easement put no additional burden on the servient estate. Easement holder has right to use reasonable efforts to clean or repair an easement. If repairs or cleaning is needed, a secondary easement is held by the dominant estate for doing what is reasonably necessary to complete the job. The secondary easement gives the easement holder a right to use additional land on the servient estate. Servient estate holder has the right to make reasonable improvements to his property containing the easement if the improvement does not unreasonably interfere with the right of passage for the easement holder. Generally, easements cannot be moved by unilateral act: Courts will allow an easement to be moved when the parties mutually and expressly agree to move it. Some courts will allow movement when there are strong reasons to move it. Easement NOT for the exclusive right of easement holder. An easement holder can allow others to use the easement when there is no express provision on exclusive use. Maintenance: Easement holder has duty to maintain easement: NOT servient estate holder. No material alteration in nature of easement allowed: In Triplett v. Beuckman, the court held that the easement holder was not entitled to tear down a bridge over which plaintiff reserved an easement and build a causeway leading from the mainland to the island. The easement holder made a material alteration to the easement and to the nature of the property (the servient estate) because the lake was circular and by building a causeway, the lake became impassable where the causeway was erected.

Transferability: Easements appurtenant are transferable but easements in gross (which are for someones personal benefit) cannot be transferred in general. There are two exceptions that some courts make for easements in gross: Parties intend to make easement in gross assignable: o In Miller v. Lutheran Conference, it was evident from the deed that the parties intended to make the easement granted to Frank Miller assignable because the language of the deed conveyed a privilege to Miller and his heirs and assigns.

Divisibility: If there is a division of the easement, the easements must be used or exercised as an entirety. In Miller, the court found that the two easement holders jointly exploited the privileges of the easement to fish and boat on the lake, and that the proceeds were simply to be divided by their respective interests in the easement. When one of the easement holders conveyed a license to defendant camp to use the lake, he was in violation of his rights in the easement because the grantees had to exercise their privilege jointly as one stock.

Termination: Mere non-usage of easement does not amount to abandonment: An easement holder must show a clear intention to abandon his right to an easement in order for the easement to be terminated (Lindsey v. Clark) Express Easement may contain a provision for its own expiration. Easement implied from necessity ends when necessity ends. Easement may be released by its holder. Easement ends by merger when title to both servient tenement and dominant tenement come into the same hands. Easement may be terminated by prescription if the owner of servient tenement obstructs the easement area for the prescriptive period.

Easement is extinguished by estoppel if the servient estate holder changes his position in reasonable reliance on the representations made or conduct by the owner of the easement.

Easement terminated by destruction of a building that serves as the dominant or servient tenement.

Easement extinguished when holder of the dominant tenement misuses the easement in such a manner that it is impossible for a court to enjoin the misuse.

Easement ends when servient estate is conveyed to a bone fide purchaser without actual, constructive, or inquiry notice of the easement.

Easement ends when servient tenement is condemned by the government under the power of eminent domain.

Implied Dedication: An easement from implied dedication can only be established when the owners intention to dedicate property for a particular use is clearly manifest.

Custom: An easement will be implied by custom only when use of property for a particular purpose has continued from time immemorial. Many courts recognize this doctrine.

Public Trust: The doctrine of public trust applies when the state holds a resource that is available for the free use of the general public. Courts will enjoin any governmental conduct to relocate the resource to more restricted uses or to subject public uses to self-interest of private parties.

Licenses in land: Licenses privilege their holders to go upon the land of another. But unlike an easement, a license is not an interest in land; it is merely a privilege, revocable at the will of the licensor. A license is personal to the licensee and, thus, inalienable. Any attempt to transfer a license results in revocation by operation of law. Revocability: In contrast to an easement, a license can be revoked at any time even when the licensee has not done anything wrong. The Restatement provides three situations in which the license cannot be revoked: Licensee must be given reasonable opportunity to remove himself from land.

License coupled with an interest can be terminated only to such an extent as not to prevent the license from being effective to protect the interest with which it is coupled. I.e., a license coupled with an interest is irrevocable as long as the interest lasts. For example, a vendee of chattel may enter sellers land to remove the chattel, and a future interest holder may enter and inspect land for waste.

Expenditures of capital or labor are made in exercise of license in reasonable reliance upon representations of licensorlicensee is privileged to continue use permitted by license to the extent reasonably necessary to realize upon his expenditures. I.e., if a licensee invests substantial amounts of money or labor in reliance on the license, the licensor is estopped to revoke. The license becomes an easement by estoppel, which lasts until the holder receives sufficient benefit to reimburse him for his expenditures. See C. Irrevocable License.

In Mosher v. Cook, the court held that a grocery store was entitled to ask a customer to leave for writing down prices of store items because the customer did not possess an irrevocable license to remain on the stores property, even though the stores revocation of license seemed unreasonable.

Lease v. License: In a lease, the lessor has an interest in the property which gives him exclusive right to possession; in a license, there is no interest in the property and the licensee merely has the privilege to do some act on the land. In Linro v. Westage, plaintiff had an agreement with a condominium owner to operate laundry facilities on each floor of the condo building. The agreement had been termed a lease, so when defendant cancelled plaintiffs contract before the expiration date of the contract, P sued. The court held that the agreement was not a lease but a license because the intent of the parties showed that plaintiff did not have exclusive control and dominion over the designated space, evidenced by his limited access to the building. Because the agreement was a license, defendant was entitled to revoke the license at any time.

Irrevocable License: An irrevocable license is called an easement by estoppel. Courts are split over this doctrine, some holding that the doctrine applies where a licensee has spent much time and money on the license, and others holding that a license by nature is revocable. Courts that adhere to the irrevocable license doctrine hold that the license is valid as long as it is necessary. In Stoner v. Zucker, the court held that Ps grant of a license to D to build an expensive drainage system on his property for Ds use was irrevocable because D has expended enormous amounts of labor and money on the ditch.

Real Covenants: o A real covenant, normally found in deeds, is a written promise to do something on the land (an affirmative covenant, e.g., maintain a fence) or a promise not to do something on the land (a restrictive covenant, e.g., not build multi-family dwelling). Real covenants run with the land at law, meaning that subsequent owners may enforce or be burdened by the covenants. o Covenant vs. Equitable Servitude: Covenant: Intent to bind successors Touch and concern Privity of estate Horizontal Vertical

Equitable servitude: Intent to bind successors Touch and concern Notice

Creation: No covenant will run with the land unless it is also an enforceable promise as between the original parties. Thus, it must not violate any positive rules of contract or constitutional law. Because a covenant that can run with the land concerns land, most courts hold it is subject to the Statute of Frauds and must be in writing and signed by the original promisor. A minority rule eliminates this requirement on the ground that the covenants themselves are not interests in land, but merely refer to land. Thus, in

these minority jurisdictions, it may be possible to enforce orally expressed covenants. Reciprocal Negative Easement: An equitable servitude is a covenant that, regardless of whether it runs with the land at law, equity will enforce against the assignees of the burdened land that have notice of the covenant. The usual remedy is an injunction. Generally, equitable servitudes are created by covenants contained in a writing that satisfies the Statute of Frauds. There is one exception: Negative equitable servitudes may be implied from a common scheme for development of a residential subdivision. Thus, if a developer subdivides land, and some deeds contain negative covenants while others do not, the negative covenants will be binding on all parcels provided there was a common scheme of development and notice of the covenants. In Sanborn v. McLean, the court found that a reciprocal negative easement was attached to defendants land because a common owner conveyed many other lots in the subdivision with express restrictions. Even though there was no express restriction in defendants deed, the restrictions on the neighboring lots provided the basis for the negative easement, and D had constructive notice of this easement because all other lots in the subdivision contained only residential homes. Upon noticing the residential character of the neighborhood, D should have inquired about restrictive covenants on his land, and he would have learned that he could not build a gas station at the back of his lot. o Enforcement at Law Against Subsequent Purchasers: Contract law governs the relationship between the original parties to a real covenant. But there are separate concerns for enforcing the covenants against subsequent purchases. Thus, for a covenant to run with the land at law, the following five requirements must be met. The covenant must be enforceable The meaning must be CLEAR. Aesthetic covenants are generally enforceable where purpose and intent can be ascertained.

In McHuron v. Grand Teton Lodge, the court held that approval for fiberglass shingles was not unreasonably withheld where a clause in a deed allowed for a committee to approve all building plans so that the architecture would blend with the natural surroundings. Underlying the courts opinion was an unwillingness to involve the judicial branch in determinations of aesthetic appropriateness.

The original parties must have intent that covenant will run with the land: The original parties must intend that their successors be bound by its terms; intention ascertained by wording of covenant and surrounding circumstances. In Runyon v. Paley, the court found that a restrictive covenant allowing only two residential homes to be built on a piece of property was a real covenant that ran with the land and not a personal covenant intended only to benefit the original covenantor because the language was clear in the deed that the original owner wanted to preserve the residential character of the land.

The covenant must Touch and Concern the land: The covenant must directly relate to the land in some way and not concern a collateral matter of a personal nature. In Runyon, the court found that the servient estate was clearly touched by the covenant because restrictions were placed on the use of the land, and that the dominant estate was also touched because the restriction affected the value of the dominant estates land. Annual Charges on property owners in a subdivision: Where a deed for property in a subdivision calls for an annual payment by property owners to maintain public areas in the subdivision, the requirement of touch and concern is met even though the covenant does not specifically touch the land owned by the property owner. Because the covenant enables all property owners to have

an easement in common enjoyment of the land, the covenant does touch and concern the land. This was the approach taken by the court in NePonsit Property Owners Assn v. Emigrant. Privity of Estate: There are two kinds of privity of estate, and most jurisdictions require that both of these types be satisfied for privity of estate to exist: o Horizontal privity: Refers to a land transaction between the covenantor and covenantee and is established by a conveyance from one party to another at the time the covenant is made. A minority of jurisdictions require the covenantor and covenantee each hold a continuing interest in the property covered by the covenant. o Vertical privity: Refers to a land transaction between an original party and that partys successor. It is universally recognized that vertical privity between the original covenantor and the covenantors successor must exist in order for the burden of the covenant to run with the land at law. Problems arise under this requirement only when the covenantor transfers and estate less than the covenantor owns. In such case, the individual receiving the lesser estate is not bound by the covenant. Distinguish this situation from the case where the covenantor transfers a geographical portion of the burdened estate. There, the purchaser takes subject to a proportionate burden of the covenant. o In Runyon, horizontal privity was established by the record, which showed that when the land was originally conveyed, the person to whom the land was conveyed covenanted to use the property for the purposes specified in the deed. Vertical privity was established between one of the plaintiffs and

defendant and their predecessor because the each succeeded to the estates held by covenantor and covenantee. Plaintiff Runyon, however, did not establish vertical privity because he did not succeed in any interest in land held by the original owner at the time the covenant was created, but acquired interest prior to the creation of the covenant. Because Runyon was not party to the covenant, he is not in privity with the original parties and may not enforce the covenant as a real covenant running with the land at law. o Homeowners Associations: Homeowners assn does not own any land, so, technically, they cannot be in privity of estate with another party. However, courts have viewed corporate plaintiffs as agents of property owners and have held that in substance, there is privity of estate between the assn and a particular property owner in the subdivision. This approach was taken by the NePonsit court. Notice: The covenantors successor must have notice of the covenant to be bound by it. Under modern recording acts, to be bound by a covenant, a subsequent purchaser for value must have had actual, inquiry, or record notice of the arrangement at the time of purchase. o Enforcement in equity against subsequent purchasers (equitable servitude): The assistance of equity is required whenever the plaintiff desires injunctive rather than monetary relief. In those situations, courts may dispense with some or all of the legal requirements. For a covenant to run with the land as an equitable servitude, it is generally only required that the particular burden or benefit involved touch and concern land. Horizontal and vertical privity are not required. However, equity requires that the party to be burdened by a covenant has had notice of it. Thus,

equity courts eliminate privity of estate and focus on reasonableness, intent, and notice. The court in Tulk v. Moxhay took this approach and held that a subsequent purchaser of land could not break a covenant not to develop a garden area on the property. In this case, Tulk sold Leicester Square in London to Elms, who promised for himself, his heirs and assigns not to build on the Square. Elms conveyed the Square to Moxhay, who had notice of the covenant. Moxhay proposed to build on the Square, and Tulk sued for an injunction. The chancellor granted the injunction, and deemed it inequitable that a covenant should be unenforceable against a subsequent purchase where the purchaser acquired the land with knowledge of the restriction. Moxhay probably supposed himself bound by the covenant and probably paid less for the land than he would have had to pay for unburdened land. To hold the covenant unenforceable would give Moxhay an advantage he did not bargain for and would unjustly enrich him. o Who may Enforce: Developers: Courts are split over whether or not to allow a developer to enforce restrictive covenants where the developer no longer has an interest in the lots covered by the restrictions. Some courts allow the developer to enforce a restrictive covenant, especially where the developer owns nearby land, because he retains a property interest in the enforcement of the covenant. In Christiansen v. Casey, the court did allow a developer who owned nearby land to enforce a restrictive covenant concerning the type of fence that could be erected in a subdivision. Other courts have not allowed developers to enforce these covenants even when the developer owned adjacent property.

IV. THE MODERN REAL ESTATE TRANSACTION Introduction: Overview and Real Estate Brokers o Real estate brokers: Brokers are persons licensed to negotiate sales, leases, financing and related real property transactions. Without a license, no person may claim

compensation for performing the above services, except for a finder's fee where no more is done than introducing the parties to one another. Persons working under a broker are usually called salespersons and must also be licensed. Employment agreement: Relationship between broker and purchaser is less formal than relationship between broker and seller Three kinds of listing agreements: Exclusive right to sell: o o Most frequently used Seller pays broker commission regardless of who finds buyer Exclusive agency: o Requires seller to pay broker commission if anyone but the seller produces a purchaser Open: o Obligates seller to pay commission only if broker produces buyer o Commission: Majority Rule: Broker entitled to commission when he produces a buyer ready, willing, and able to purchase on the sellers terms, even if the sale does not go through. Minority Rule: Broker does not earn a commission until the sale is consummated. NOTE: A court may hold that the broker is entitled to a commission if the seller interfered with the closing in bad faith or otherwise was at fault for the sale not being consummated. This was the rule followed by the court in Tristrams Landing v. Wait where the court held that when the buyer refused to go through with the purchase, the broker was not entitled to commission because ordinarily this is what sellers expect when they sign listing agreements and generally the commission comes out of the proceeds of the sale. Relationship between brokers and lawyers: Seller may concurrently employ multiple brokers

Brokers can practice law in some ways if it is in public interest: In doing such things as filling out earnest money agreements, brokers are practicing law. But it is in public interest to allow them to do so because it is efficient and inexpensive, where hiring a lawyer would be inefficient and costly. In Cultum v. Heritage House, the court held that a broker was permitted to practice law in filling out an earnest money agreement because it was efficient and the public was protected because the broker had to be licensed.

Duty to comply with standard of care of an attorney: When a broker engages in the practice of law, she has the duty to obey all reasonable instructions and directions given by principal, and adhere faithfully to them. In Cultum, the broker did not obey her clients instructions because she failed to include a subjective right in her contingency clause.

NOTE: There are concerns with a broker engaging in the practice of law, however, because a broker often works under a dual agency arrangement, representing both seller and buyer in the same transaction. This is something an attorney would not do.

Brokers duty to seller and buyer: Broker must exercise good faith and loyalty in representing the principal. Breach of fiduciary obligation might result in loss of right to commission. Some jurisdictions require broker to disclose known defects to a potential purchaser. Various bases for a brokers duties: Contract. Must perform any obligations undertaken by virtue of the terms of listing contract. Agency. Must show loyalty, integrity and good faith as an agent to the principal; Malpractice. Must meet the standards of due care expected of a professional in this field; Licensing. Must comply with the duties imposed by virtue of the brokers license.

Real Estate Contract of Sale o Two kinds of contracts for sale of land:

Standard real estate sales contract Installment land contract

Statute of frauds: Necessary: Writing required In order for a writing to satisfy the statute of frauds, the writing must establish the essential terms of the contract, including names of the parties, description of the land, and the purchase price, and the terms must be such that neither party can reasonably misunderstand them. Additionally, the contract must be signed by the party to be charged. Part performance The statute will not be enforced to work a fraud or hardship on one who has acted in reliance on an oral agreement. Payment of consideration, taking possession of the property and making substantial improvements upon it are both critical acts. In Cash v. Maddox, the court held that a check containing both parties names was too vague and indefinite to satisfy the statute of frauds because there was no description of the land or evidence of the total purchase price. The dissent disagreed, and reasoned that because there was a written memo (the check), which showed an agreement for sale of the land, the statute did not apply.

Marketable title: A purchaser is entitled to a marketable title; thus, every contract contains an implied warranty that the seller will provide marketable title (title reasonably free from doubt) at closing. It need not be perfect title, but must be free of questions that present an unreasonably risk of litigation. NOTE: A vendor has no obligation to have a marketable title prior to the date set in the contract for performance. His title must be marketable only at time he is obliged to tender a deed, i.e., at closing. Reasonable Prudent Buyer standard: A title is not marketable if a reasonable prudent buyer would not buy the property with the cloud on the title. A cloud sufficient to prevent a reasonable person from buying the land would be if a

person other than the seller held an interest in the land, or if the vendor owns less than he has contracted to convey, or if there are irregularities in his chain of title, or if the property is subject to an encumbrance. Land use ordinances and physical defects in the property do not affect title, and monetary encumbrances may be removed by using part of the purchaser's funds to satisfy them. Beneficial easements, if visible or known to the buyer, do not impair the marketability of title. If title is unmarketable at the time for closing, the purchaser may withdraw or sometimes be granted specific performance with an abatement of the purchase price. In G/GM v. Susse Chalet Motor Lodge, the court held that a title burdened by a lease that was not binding on the purchaser did not make the title unmarketable because the buyer was not prevented from obtaining title insurance, and a reasonable person would have still bought the property even with the defect on the title. The court found that the buyer was merely trying to get out of the sale, and was using the title defect as an excuse. Written Agreement for Cloud-free Title: If parties expressly agree that a sale is contingent upon a cloudfree title, then any defect in the title is a sufficient excuse for not going through with the sale. Sellers breach for unmarketable title: Seller aware of title defect: Ordinarily, a seller is not relieved of liability for a breach of a promise to sell property simply because he turns out not to have marketable title if he knew or should have known that he did not have marketable title. While the buyer cannot obtain the property, the buyer is entitled to damages based on the contract-market differential. In Warner v. Denis, the husband should have known that his title to land was not marketable because his wife was a joint tenant. When he was unable to obtain his wifes consent for the sale, he was still liable for damages to the buyer. Seller unaware of title defect: Courts are split over whether a seller who honestly believed he had marketable title should be

liable to a buyer for actual damages when the title turns out to be unmarketable. o Specific performance: All real property contracts are specifically enforceable because real property is unique by nature and damages at law are an inadequate remedy. In Pruitt v. Graziano, the court held that a condo unit was unique and the buyer was entitled to specific performance even though all condo units looked the same. Also, this particular condo had some remodeling done to it. (Courts will also give sellers specific performance.) o Equitable conversion risk of loss: Once a binding, specifically enforceable contract has been executed, the purchaser becomes the equitable owner of the property and the seller holds legal title only as security for payment. Following this doctrine, many states hold that the purchaser thereafter takes the risk of innocent destruction of the premises. Other courts imply a provision in the contract that the purchaser will receive the property in the condition it was in when the contract was executed, thereby allocating the risk of loss to the seller (or sometimes until the purchaser takes possession). The parties may independently insure themselves against risk of loss or may contract between themselves as to how the risk should be allocated. Majority Default RuleRisk of loss placed on buyer: This rule, known as equitable conversion, is justified on the grounds that the buyer gains both the benefits and burdens when he takes an interest in the land. Minority Default RuleSeller bears risk of loss: Some courts place risk of loss on seller, absent a contractual provision to the contrary, because there is an implied condition that the buyer is to get the property in the same condition it was in at the time of contracting, i.e., without damages. Uniform Vendor-Purchaser Risk Act: Party in possession of property bears risk of loss when contract does not address risk of loss. Special Agreement: None of the above rules apply when the parties have specifically provided for risk of loss allocation in their sales contract. Loss is determined according to the above doctrines only when the contract is silent on the matter. Bryant v. Willison Real Estate Co. Financing

Mortgages: A mortgage is a promise made by a property owner (the mortgagor or borrower) to a creditor that, if he does not perform some specified obligation (e.g., paying off the loan), then the creditor (the mortgagee or lender) may utilize the owners property to satisfy the obligation. In other words, the property is put up as a security on the loan. A mortgage transaction generally involves two documents. One document memorializes the basic obligation (usually to pay, represented by a promissory note). The second document (the mortgage) provides that the mortgagee/lender may reach some property of the person signing this document, the mortgagor/borrower, (who usually is the obligor under the first document) if the prime obligation (payment) is not performed. A mortgage is always a security for some other obligation and is secondary to it. The mortgage transaction consists of: (i) the note, which evidences the debt; and (ii) a mortgage, which is an agreement that the land will be sold if the debt is not paid (security for lender). The note and mortgage may be one document. Usually, the mortgage is recorded giving the lender priority over subsequent purchasers. Underlying Theory and Timeline: When a buyer takes a mortgage on property, the buyers interest is a fee simple absolute subject to a security interest in the lender. The lender has the right to foreclose on the property if the buyer defaults. The following is a timeline of a mortgage arrangement:

Commitment Application from Lender

Closing mortgage executed

Payoff (15-30 yrs); If buyer cannot pay off mortgage, equity of redemption period begins in which buyer is allowed more

Foreclosure; If buyer fails to pay off mortgage in equity period, lender forecloses.

Statutory Redemption Ends; if debtor fails to pay foreclosure price, he loses the land

After foreclosure, some states have

time to pay off debt

statutory redemption period in which debtor is given chance to buy back land at foreclosure price.

Financing

Default

Foreclosure Post-Foreclosure

Buyer finances purchase of land using the land as collateral. Usually done by giving lender a mortgage on the property.

Mortgagor/borrower defaults. Mortgagee/lender has right to foreclose. Up until the foreclosure sale, borrower may redeem by paying off mortgage and accrued interest. (Equitable redemption).

Foreclosure must be by sale, usually judicial sale. If proceeds of sale insufficient to satisfy debt, mortgagee/lender can Proceeds distributed according to priorities of security interests. bring personal action against borrower for deficiency.

About half the states give borrower a right to redeem for a fixed period (e.g., six months) after foreclosure by paying sale price. (Statutory redemption).

Foreclosure: When a mortgagor/borrower defaults, the mortgagee/lender does not acquire an interest in the property, but may terminate the mortgagors interest in the land by foreclosing. Almost all states require foreclosure by sale, under which the property is sold to satisfy the debt in whole or part. Any surplus from a foreclosure sale goes to the mortgagor; conversely, the mortgagor may owe a deficiency judgment if the sale does not produce enough. NOTE: The debtor can bid at the foreclosure sale. The majority of states allow the debtor to take the remaining proceeds from a foreclosure sale after the lender takes the value of his debt. If there are two lenders involved, then the 1st lender gets the total amount of

the debt owed to him, and the 2nd lender gets the debt owed him only when the 1st lender has taken all that he is owed. If there are not sufficient funds to cover the debt, the second lender can get a deficiency judgment against the debtor and go after the debtors other assets. Redemption: Redemption in EquityAt any time prior to the foreclosure sale, the mortgagor may redeem the property by paying the amount due. If the note or mortgage contains an acceleration clause, the full balance of the note or mortgage must be paid to redeem. This right cant by waived by the mortgage. Statutory RedemptionAbout half the states allow the mortgagor to redeem the property for some fixed period (e.g., six months) after the foreclosure sale has occurred. o Subject to Financing Clause: Financing first satisfied but then fails after voluntary action undertaken by buyer: In such a case, the buyer is not entitled to a refund of deposit because the intent of the financing clause is to protect a buyer from involuntary breach. In Bruyere v. Jade Realty, the buyer was not entitled to a refund of deposit when the financing was first obtained but then failed due to a divorce. The court held the divorce was voluntary act of buyers. Failure to specify interest rate: Courts are divided on whether or not to enforce a contract for sale where an interest rate is not included in the financing clause. Most courts hold that the contract is too vague and indefinite to be enforceable (approach followed in Holmer v. Malas), but other courts have been willing to read a reasonable rate into the financing clause, and some courts have held that current rate is definite enough to enforce the contract. Good Faith Effort required: Buyer must make reasonable good faith efforts to obtain acceptable financing. Taking over an existing mortgage: When a buyer takes over an existing mortgage, he can either start paying without any obligation to the lender, in which case he will not be liable for a deficiency; expressly assume the obligation, in which case he will be liable for a deficiency. Recording systems:

At common law, if a grantor conveyed the same property twice, the rule was first in time, first in right. This meant that the first person the property was transferred to (not the first who recorded) had superior rights. The recording statutes were a response to this system and change the outcome under certain circumstances. Recording is not essential to the validity of the deed between the grantor and grantee, but can be essential to protect the grantees rights.

The recording statutes provide, generally, that (1) any document affecting title may (should) be recorded; (2) by recording the document, constructive notice of its existence is given to the public; and (3) if the document is not recorded, the person claiming under it will lose priority as against subsequent bona fide purchasers.

Recording statutes: There are three types of recording statutes, but under all three, the burden is on the subsequent purchaser to prove that he qualifies for protection under the statute (i.e., it is the subsequent purchases who has to fit into the recording system). Of these, almost all states follow notice or race-notice: Race: Whoever records first wins. Notice is irrelevant. Notice: A subsequent purchaser who pays value and takes without notice of a prior unrecorded instrument is protected and prevails. Key is that the subsequent purchases had no actual or constructive notice at the time of the conveyance. Ex: O conveys to A on January 1. A does not record. O conveys to B on January 15 for valuable consideration. B has no notice of the conveyance to A. B prevails over A. It is irrelevant whether A recorded after January 15 and before B recorded, because B had no notice at the time he took. Race-Notice: A subsequent purchaser is protected only if she takes without notice AND records before the prior grantee. Ex: O conveys to A on January 1. A does not record. O conveys to B on January 15 for valuable consideration. B has no notice of the conveyance to A. A records on January 18. B records on January 20. A prevails over B because B did not record first.

Who is protected:

Generally, only bona fide purchasers are protected by the recording system. In order to qualify as a bona fide purchaser, one must give valuable consideration and take without notice.

Purchaser for Valuable Consideration: Donee not protected under statute: In Anderson v. Anderson, a grantors children were not protected under the statute even though they recorded first because they were donees and had not paid valuable consideration for the land. (However, donees can protect themselves against subsequent purchasers if they record first). Substantial Value v. Nominal Consideration: Courts are divided on whether or not a person must give substantial value or nominal consideration to be protected under the statutes. Some courts hold that nominal consideration is enough to make the grantee a purchaser for value under the recording statutes, while others hold that the grantee must pay substantial value. Recorded judgment lien takes precedence over unrecorded prior conveyance if statute so provides: In Rowe v. Schultz, Rowe was granted land prior to a judgment lien being attached to the land. However, Rowe was not entitled to title to the land because he did not record before Schultz, who obtained the judgment lien, recorded.

Without Notice: Notice must be available at time that value is given for the deed: If A learns on his way to the courthouse to record his deed that the property he bought was actually conveyed to B earlier but not recorded, A does not have notice of the prior conveyance and is protected under the recording statute. If A had purchased on an installment land contract, A would still be protected under the statute, but would not have total protection because A has not paid much for the land on the front end. Deed given to clerk but not recorded in index: Courts are divided. Some courts hold that once a document is filed with the clerk, the party has no further duty to inquire into the filing of the deed because requiring the filer to double-check would be impractical. Handing the document to the clerk suffices to impart

constructive notice to everyone, and the purchaser is protected against all subsequent purchasers. Other courts have held that filers do have a duty to inquire further with the clerk because they are the only ones who can make certain that indexing is done properly. If the deed is not properly indexed, then no one is on constructive notice The court in Haner v. Bruce took the first approach, where plaintiff filed an attachment lien on defendants property but the clerk failed to record the deed. Later, defendant conveyed the property to another party who had no way of knowing that the lien had been attached to the property. The court held for plaintiff, holding that mere filing with the clerk is enough to put all subsequent purchasers on constructive notice.

Establishing Title by Adverse Possession:


o In order to establish a claim of adverse possession, the possession must be: Exclusive Actual and uninterrupted Open and notorious Hostile and under a claim of right made in good faith Hostility = imports that the claimant is in possession as owner, in contradistinction to holding in recognition of or subordination to the true owner => treat the land as his own o A possessor of land, even though not an owner, may protect that possession against everyone in the world except the true owner. Although the true owner may bring ejectment against a person wrongfully possessing his property, his failure to do so in time, means that the possessor is then protected from ejectment by anyone, stranger or former owner. At this stage the possessor becomes, for all practical purposes, the owner. o Completion of a successful adverse possession gives the possessor an original title to the property, although it may not be marketable until so established in court. Such a title is only good as against those rights in the property that are extinguished by the statute of limitations; this does not include holders of future interests or non-possessory interests. Nor, generally, is the government barred by the doctrine of adverse possession. o Elements:

Actual and Exclusive: An adverse possessor will gain title only to land he actually occupies. In some cases, actual possession of the entire parcel claimed is not necessary. If an adverse possessor actually occupies a reasonable portion of the parcel, and his occupation is under color of title to the entire parcel, then he will be deemed to have constructively possessed the entire parcel, with the same result as if he had actually occupied the entire parcel. Actual possession can also include use, rather than strict possession. A party must merely use the property as an owner would use it. He does not have to build a house or use the property for other purposes for which the property might be suitable. Exclusive means that the possessor is not sharing with the true owner or the public. Two or more people may obtain title by adverse possession; they take title as tenants in common. In Jarvis v. Gillespie, the court held that possession is gauged by the actual state of the land and not with reference to its capability of being used for another purpose. Ps use of land as a parking area, cattle grazing area, and storing area were consistent with the parcels nature and thus his possession was actual.

Continuous and Uninterrupted: The adverse claimants possession must be continuous throughout the statutory period. Intermittent periods of occupancy are not sufficient. However, constant use by the claimant is not required as long as possession is of a type that the usual owner would make. Also, there need not be continuous possession by the same person; an adverse possessor can tack his own possession onto the periods of adverse possession of his predecessors, but privity is required. Thus, the statute of limitations does not begin running again each time the property is conveyed. In Carpenter v. Huffman, the court held that continuity of possession of the land adversely held was not broken when the grantor conveyed his property to the grantee, and the two possessions could be joined and considered as one continuous possession.

Open and Notorious:

The adverse possessors occupation of the property must be sufficiently apparent to put the true owner on notice that a trespass is occurring. The purpose of this requirement is to give the true owner an opportunity to stop the adverse use. But lack of actual knowledge by an owner is no defense.

In Marengo Cave v. Ross, the court held that plaintiffs use of defendants property was not visible because the potion of defendants land that was being used was underground and Ps use of the land could not be easily discovered. As a result, the portion of the cave that was on Ds land was blocked off so that no one could access that part of the cave. Was this the best result here? Defendant did explore the cave on numerous occasions and could have had a survey conducted earlier which would have revealed Ps use of Ds property. It could be argued that Ps use of the land was visible.

Hostile: This requirement is satisfied if the possessor enters without the owners permission and uses the property as his own, including with regard to others, permitting them to be on the property only with his permission. It doesnt matter if the adverse user mistakenly thinks that the property is his own or is actually hoping to steal the title. Ouster: Where possession is undertaken permissively (e.g., by lease), possession does not become adverse until the possessor makes clear to the true owner the fact that he is claiming hostilely. If a tenant remains in possession after the expiration of the lease, he is presumed to have permission of the landlord. In Chaplin v. Sanders, defendant thought that the strip of land that he was using as a facility for his trailer park was his own. The court held that Ds subjective intent regarding his true interest in the land was irrelevant, and the only thing that mattered was that D treated the land as his own without permission.

Statutory Period: Possession must last for the statutory period. Each jurisdiction has its own statutory period for the amount of time adverse possession must last.

Title Insurance: o Title insurance is a guarantee made by a third party who has examined the chain of title, that the title is what it is represented to be in the title insurance policy. A title insurance company usually has its own set of land records and indexes, generally organized upon a tract basis. When a property is about to be sold, the company investigates the vendors title and reports on it to the purchaser so that he or his attorney may determine whether to accept it (i.e., whether it is marketable). o The insurance policy will contain exceptions to coverage for defects the insurance company finds but does not want to insure. Thus, the company can essentially assess its risk in issuing the policy very accurately. o Tort liability for negligence in searching records: Jurisdictions are split on whether or not the insured can sue the insurer in tort for failure to search title records, or searching negligently. Some jurisdictions hold that because a title insurance company holds itself out as having expertise in searching title records, plaintiffs who rely on that expertise can sue in tort when the insurer fails to search for (or find) title defects. Other jurisdictions hold that because the title insurance company does not purport to act as anything other than an insurance company, no tort liability exists unless the co. voluntarily assumes a duty to search the title in addition to contracting to insure the title. In Greenberg v. Stewart Title, the court held that the title insurance company was not liable in tort (but maybe in contract) when it failed to search the insureds title for defects and as a result, the insured could not sell his property. o Title holder must give notice to title company when he discovers a title defect: In Stewart Title, the court held that the owner could not remain silent when he learned of a title defect, sell the property with a full warranty deed, and then contend that the policy remained in effect as an owners policy despite the specific wording of the contract that the policy became a warrantors policy upon the sale of the property. Had the owner notified the company of the defect and the prospective sale of the property, the company could have mitigated an impending loss under the policy by negotiating with the gas company to remove the gas line or purchased its easement rights.

Deed:

Deeds must include the following: grantors name, grantees name, words of conveyance, grantors signature, and some states require witness signatures.

Description of land: Deed must provide proper description of land conveyed in order for transfer of title to be conveyed. There are three types of legal description: Metes and Bounds: Starts at a definite point on boundary of a tract of land and traces perimeter by reference to appropriate directions and distances back to starting point. Government Survey: Land divided into rectangular tracts with principal meridian and baseline intersection marking center of a township. Plat: map that divides subdivision into blocks and blocks into lots.

Adequacy of Description: Part of the Northeast Quarter of the Northeast Quarter of Section 9 Township 16 North Range 13 East all in Douglas County Nebraska? NOT ADEQUATE: what part of the Northeast Quarter? All lands and real estate belonging to the said party of the first part, wherever the same may be situated. ADEQUATE: this can apply only to those lands the said party owns, so no confusion. My Farm. ADEQUATE: I have only one farmbut there may be a question when you consider that the farm might have a residential area separate from the actual farming area. The lot on which the residence and Dairy-Joy or ice cream business of Kenneth L. Bennett is located. NOT ADEQUATE

Covenants of title: Covenants of title are not implied in a deed, but the grantor usually makes express covenants about the status of title to property conveyed. The express covenants merely give the grantee and grantees successors the right to recover damages from the grantor if title to the property is not as covenanted. There are three types of deeds, classified by the type of covenants given to grantee: General Warranty Deed: Seller promises that title is free from defects and includes present and future covenants: Present Covenants: these covenants are personal and do not run with the land; therefore, the remedy for a remote grantee when

the encumbrance has not been removed is against the immediate grantor, whose recourse is against his grantor. Covenant of seisingrantor promises that he owns property. He must have title and possession at the time of the grant. Covenant of right to conveygrantor promises that he has authority to convey. Title alone will not satisfy this covenant. Covenant against encumbrancesgrantor promises that land is not burdened by mortgages, liens, easements, leases, restrictive covenants, or other encumbrances. o Is a violated public zoning ordinance at time of conveyance a breach of covenant against encumbrances? Courts are split on this issue. o In Wilcox v. Pioneer Homes, the court held that a violated zoning ordinance at conveyance was a breach of covenant and awarded the grantee damages for having to purchase a strip of land to comply with the ordinance. Other courts have held that such a violation is not a breach. However, the existence of a public zoning ordinance is not a breach of covenant because the government has the right to regulate land. Existence of a private zoning restriction in a subdivision at conveyance is a breach of the covenant. Future Covenants: These covenants run with the land. Covenant of quiet enjoymentgrantor promises the grantee will not be disturbed in possession by a third partys lawful claim of title, i.e., by a paramount titleholder. o Knowledge of a potential breach of covenant does not prevent a grantee from suing for breach. In Foley v. Smith, the court held that a grantee who knew that the grantor had previously conveyed the land to another party did not prevent the grantee from suing the grantor because a

general warranty deed warrants against known and unknown defects, and grantees have the right to rely on the covenants for their protection. o Covenant of Warranty or quiet enjoyment is broken only upon actual or constructive eviction under a paramount title existing at the time of conveyance. If a grantee is not evicted or has not paid off an encumbrance but merely discovers an encumbrance on the title, he may recover only nominal damages. In Proffit v. Isley, plaintiff could only recover nominal damages when he discovered an outstanding mortgage on the property because he had incurred no expense because of it. (Statute of limitations will begin running from the point when eviction occurs.) o Interest and attorneys fees are allowable items of damage for breach of covenant of quiet enjoyment: In Foley, the court awarded plaintiff more than the actual amount of consideration paid for the interest and reasonable attorneys fees expended in good faith to defend the title. Covenant of warrantygrantor agrees to defend against reasonable claims of title by a third party, and to compensate the grantee for any loss sustained by the claim of superior title. o Covenant breached when steps are taken to enforce an encumbrance. Covenant of further assurancegrantor promises to perform acts or execute documents necessary to make the title good. Special Warranty Deed: Includes same six covenants as general warranty except that grantor promises that the estate is free from encumbrances made by the grantor, i.e., that he did not create the title defect. If the defect, such as an easement, was created by the grantors

predecessor in title grantor is not liable for breach of covenant against encumbrances. Quitclaim Deed: A quitclaim deed releases whatever interest the grantor has in the property. No covenants of title are included or implied; grantor conveys whatever interest he has in property and grantee has no recourse in the event of breach.

V. ATTRIBUTES OF LAND OWNERSHIP Right to Exclude and Freedom of Alienation: o Limits on Alienation: The right to transfer is absolute. Except when it isn't. Alby v Banc One => Alby conveyed land to niece for a very cheap price with a deed restriction that if the land was encumbered or mortgaged it would automatically revert to Alby and it gave her right of first refusal if niece wanted to sell. Niece mortgaged it for money and then defaulted. Courts don't like restraints on alienation, but restraints are valid if justified by legitimate interests of the parties. Court balanced interests of keeping it in the family (a valid interest historically) and the needs of free alienability. Court decided that the family interest justified the restraint. o Limits on right to exclude: Right to exclude is often thought of as one of the most fundamental of property rights. Shack => Defendants work for a non-profit pursuant to federal Statute, to provide health and legal services to migrant farm workers. Tedasco owns farm and houses migrant workers on his land. Tedascao tried to keep defendants out and told them that if they wanted to provide legal services, they had to meet in his office under his supervision. Court went to social policy and human values. Migrant workers are socially isolated and don't even know their rights. Common Law Maxim => one shouldn't be able to use property to insure rights of others. It is unthinkable that an employer can assert such control over employees. Property rights have to be balanced against these other rights. Government or non-profit workers cannot be prevented from entering premises to assist workers in their

quarters. This is a classic Legal Realist way to resolve things. Not formalist at all. o Open housing Limit on Transfer, Use, Possess, and Exclude Social Values can limit property rights. Shelley v Kraemer => restrictive covenants that exclude people from ownership based on race or color are contrary to 14th Amendment (equal protection under law). Court reviewed because it was a government action for courts to enforce these covenants. Normally SC can't reach private agreements. This is a limit on the Bundle of Rights

VII. GOVERNMENT CONTROL OF PRIVATE LAND USE Land Use Planning: o Common law: Nuisance Common law way to bring about land use planning o Zoning Necessary through growing urbanization Standard State Zoning Enabling Act idea was to encourage states to zone so that states could delegate its police power to local governments. To enforce land use Town council, city. Commissioners should adopt a comprehensive plan Then they should adopt zoning so land uses to conform to that plan Then they delegate authority to a local board to do that Problem: no actual plan. Ad hoc.

Police power presumed to reside in the state

Ordinance usually provides for use regulations as well as height and bulk regulations Existing uses are normally grandfathered in when zoning happens. The thought was that these businesses would wither away and die, but it really just creates an automatic monopoly

Village of Euclid => AR owned land. City implemented complex zoning scheme. AR could still use land for manufacturing, which was the plan all along, but a bunch of the neighboring land was zoned for commercial and for single-family housing. AR said that the zoning was against due process because it reduced the value of his

land by not allowing apartments nearby and by not allowing too much manufacturing. Court looked at the need to protecting interests of health and livability. This was totally legitimate. Said these decisions were closely enough related to police powers to consider them not arbitrary and capricious. So, defer to local government and consider the plan as a whole. Takings: o 5th amendment property will not be taken for public use without just compensation Judgment cannot force people alone to bear burdens which should be borne by the public as a whole Does not seem to be an express prohibition against government taking private property Property ownership as a prerequisite to liberty. Are property and liberty the same thing? o What is a taking Land use regulation was not held to be a taking When can regulations arise to the level of a taking? Not when it is to prevent nuisances or noxious uses It can when it goes too far (PA Coal v. Mahon) 3 factors (Penn Central) o o Economic interference with the claimant Interference with claimants investment-backed expectations o Character of governmental action

If there is a physical permanent occupation, a taking will be found (Loretto)

If government requires and exaction, that lacks an essential nexus with a legitimate state interest or rough proportionality of the proposed developments impact, then a taking will be found (Nolan, Dolen)

If a regulation causes a loss of all economically or productive uses of land, unless it is justified by nuisance law or other background principles of property law, it will be considered to be a taking

A regulation that would otherwise be unconstitutional absent compensation is not transformed into a background principle of a states law merely by passage of title

When can government NOT take property? Kelo v New London you can take for airports and railroads and blighted stuff. Can you take these not-blighted properties for a redevelopment plan?

Você também pode gostar